RMO 2016

Algebra Level 3

Let a a , b b , and c c be positive real numbers such that

a 1 + b + b 1 + c + c 1 + a = 1. \frac{a}{1+b} + \frac{b}{1 + c} + \frac{c}{1 + a} = 1.

What is the maximum value of a b c ? abc?


The answer is 0.125.

This section requires Javascript.
You are seeing this because something didn't load right. We suggest you, (a) try refreshing the page, (b) enabling javascript if it is disabled on your browser and, finally, (c) loading the non-javascript version of this page . We're sorry about the hassle.

4 solutions

P C
Oct 15, 2016

Relevant wiki: Titu's Lemma

The L H S LHS is equivalent to c y c a 2 a + a b \displaystyle\sum_{cyc}\frac{a^2}{a+ab} , by Titu's Lemma 1 = c y c a 2 a + a b ( a + b + c ) 2 a + b + c + a b + b c + c a 1=\displaystyle\sum_{cyc}\frac{a^2}{a+ab}\geq\frac{(a+b+c)^2}{a+b+c+ab+bc+ca} a + b + c + a b + b c + c a ( a + b + c ) 2 \Leftrightarrow a+b+c+ab+bc+ca\geq (a+b+c)^2 Using a b + b c + c a C S ( a + b + c ) 2 3 ab+bc+ca\stackrel{C-S}\leq\frac{(a+b+c)^2}{3} , we have a + b + c + ( a + b + c ) 2 3 ( a + b + c ) 2 a+b+c+\frac{(a+b+c)^2}{3}\geq (a+b+c)^2 3 2 a + b + c A M G M 3 a b c 3 \therefore \frac{3}{2}\geq a+b+c\stackrel{AM-GM}\geq 3\sqrt[3]{abc} a b c 1 8 = 0.125 \Leftrightarrow abc\leq\frac{1}{8}=0.125 The equality holds when a = b = c = 1 2 a=b=c=\frac{1}{2}

+1 Your solution is a great read:

  1. Thanks for pointing out when equality holds. Given the numerous inequalities used, we have to check that this upper bound can indeed be achieved.
  2. Be careful with trying to lump too many steps together. It is better to explain first why 3 2 a + b + c \frac{3}{2} \geq a+b+c , before moving on with the rest of the solution.

Thanks for contributing and helping other members aspire to be like you

Calvin Lin Staff - 4 years, 8 months ago

Given :

a 1 + b + b 1 + c + c 1 + a = 1 \frac{a}{1+b} + \frac{ b}{1+c}+ \frac{c}{1+a} = 1

Taking LCM and cross multiplying we get:

a ( 1 + c ) ( 1 + a ) + b ( 1 + b ) ( 1 + a ) + c ( 1 + b ) ( 1 + c ) = ( 1 + a ) ( 1 + b ) ( 1 + c ) a\cdot(1+c)\cdot(1+a) + b\cdot(1+b)\cdot(1+a) + c\cdot(1+b)\cdot(1+c) = (1+a)\cdot(1+b)\cdot(1+c)

After simplifying and cancelling the like terms on both sides we get:

1 + a b c = a 2 + b 2 + c 2 + a 2 c + b 2 a + c 2 b 1+ abc = a^{2}+ b^{2} + c^{2}+ a^{2}c + b^{2}a + c^{2}b

Using AM GM inequality separately on the terms: ( a ² + b ² + c ² ) ( a² + b² + c² ) and ( a ² c + b ² a + c ² b ) (a²c + b²a + c²b) we get:

1 + a b c 3 ( a b c ) 2 / 3 + 3 a b c 1 + abc \ge 3(abc)^{2/3}+ 3abc

Equality occurs when a = b = c = 1 / 2 a = b = c = 1/2

Now, cubing both sides of inequality we get

( 1 2 a b c ) ³ 27 a ² b ² c ² (1 - 2abc )³ \ge 27a²b²c²

Let a b c = t abc = t

1 8 t ³ 6 t + 12 t ² 27 t ² 1 - 8t³ - 6t + 12t² \ge 27t²

8 t ³ + 15 t ² + 6 t 1 0 8t³ + 15t² + 6t - 1 \le 0

The above expression can also be written as:

8 t ³ + 16 t ² + 8 t t ² 2 t 1 0 8t³ + 16t² + 8t - t² - 2t - 1 \le 0

8 t ( t ² + 2 t + 1 ) 1 ( t ² + 2 t + 1 ) 0 8t ( t² + 2t + 1) - 1( t² + 2t + 1) \le 0

( 8 t 1 ) ( t + 1 ) ² 0 (8t - 1)(t+1)² \le 0

So 8 t 1 0 8t - 1 \le 0 as ( t + 1 ) ² > 0 ( t + 1)² >0

8 t 1 8t \le 1

8 a b c 1 8abc \le 1

a b c 1 / 8 = 0.125 abc \le 1/8 = 0.125

Hence the maximum value of a b c abc is 0.125 0.125

Try my problem ' A cubic inequality'

Abhishek Sinha
Jul 27, 2018

Using the Cauchy-Schwartz inequality, we have ( ( 1 + b ) + ( 1 + c ) + ( 1 + a ) ) ( a 1 + b + b 1 + c + c 1 + a ) ( a + b + c ) 2 = a + b + c + 2 ( a b + b c + c a ) . \bigg((1+b)+(1+c)+(1+a)\bigg)\bigg( \frac{a}{1+b}+\frac{b}{1+c}+\frac{c}{1+a}\bigg)\geq \big(\sqrt{a}+\sqrt{b}+\sqrt{c}\big)^2=a+b+c+2(\sqrt{ab}+\sqrt{bc}+\sqrt{ca}). Using the AM-GM inequality on the RHS, we have from the above 3 + ( a + b + c ) a + b + c + 6 ( a b c ) 1 / 3 , 3+(a+b+c)\geq a+b+c+ 6(abc)^{1/3}, which implies a b c 1 8 . abc\leq \frac{1}{8}. The equality is achieved when a = b = c = 1 2 . a=b=c=\frac{1}{2}.

Can you please increase the font size of the indices, so that it makes it easy to read them? Thanks.

Anu Radha - 2 years, 7 months ago
Suresh Jh
Feb 6, 2018

From AM,GM we can easily Conclude that maximum value Of abc, Will be when a,b,c are are equal. Now take a= b=c and Solve it.

How do we know that fact ?

Srinjoy G - 3 years, 2 months ago

0 pending reports

×

Problem Loading...

Note Loading...

Set Loading...